If A Force F = 3i + 2j - 4k Is Acting At The Point P(1, -1, 2), Then The Moment Of F About The Point Q(2, -3, 3) As

Previous   /   Next
Claire 4 Years Ago

1 Answers

Your Answer

Connect With Answers

We are available 24x7 to answer.



+ =


Notification

Your post is submitted for moderation. It will be updated soon.